Q1. Sketch the graph of the function y = x3 – x2 - 8x by finding intercepts, intervals of increasing/decreasing, local maxima/minima, intervals of concavity up / down and inflection points.

Answers

Answer 1

Graph can be sketched on the basis of below points:

1) Intercepts

2) intervals of increasing and decreasing

3) local maxima and local minima

4) Intervals of concavity up or down

5) Inflexion points .

Given

Polynomial:

x³ – x² - 8x

Now,

1)

Intercepts:

For calculating y intercept of the polynomial,

y = f(0)

y = 0

Hence the y intercept will be (0,0)

For calculating x intercept:

x³ – x² - 8x = 0

x(x² -x -8) = 0

x = 0

x = (1 ± √33) / 2

2)

For intervals of increasing and decreasing check the derivative of function:

If f'(x) > 0 the function will be increasing

If f'(x)< 0 the function will be decreasing

Here,

f'(x) = 3x² -2x - 8

3)

Local maxima and local minima:

f'(x) = 0

3x² -2x - 8 = 0

x = 2

x = -4/3

Second derivative test:

f''(x) = 6x - 2

At,

x = 2

f''(x) = 10

x = -4/3

f''(x) = -10

Hence point x = 2 is the point of local minima and point x = -4/3 is a point of local maxima .

4)

Inflection points :

f''(x) = 0

6x - 2 = 0

x = 1/3

To check x = 1/3

Put

x = 0

x = 1

f''(0) = -2(negative)

f''(1) = 4(positive)

Hence proved .

Know more about functions,

https://brainly.com/question/31062578

#SPJ4

Q1. Sketch The Graph Of The Function Y = X3 X2 - 8x By Finding Intercepts, Intervals Of Increasing/decreasing,

Related Questions

Use the leading coefficient test to determine the end behavior of the graph of the given polynomial function. f(x) = 2x5 + 6x² + 7x³ +3 O A. Rises left & rises right. B. Falls left & rises right. C. Falls left & falls right. D. Rises left & falls right. E. None of the above.

Answers

The end behavior of the graph of the polynomial function [tex]f(x) = 2x^5 + 6x^2 + 7x^3 + 3[/tex] is described as follows: The graph rises to positive infinity as x approaches negative infinity and rises to positive infinity as x approaches positive infinity that is option A.

The leading coefficient of the polynomial function is [tex]2x^5[/tex], which is positive.

According to the leading coefficient test, if the leading coefficient is positive, then the end behavior of the graph is as follows:

As x approaches negative infinity, the function rises to positive infinity.

As x approaches positive infinity, the function also rises to positive infinity.

To know more about polynomial function,

https://brainly.com/question/10467145

#SPJ11








Please state the range for each of the following. Sketch a graph of the function sin(x-45°) +2.

Answers

The function is given by f(x) = sin(x-45°) + 2. We are required to determine the range of this function and sketch its graph. Here's how we can do it:

Range of f(x),The range of the function f(x) is given by the set of all possible values of f(x). Since the sine function can take values between -1 and 1, we have :f(x) = sin(x-45°) + 2 = [-1, 1] + 2 = [1, 3]Therefore, the range of the given function is [1, 3].

Graph of f(x):To sketch the graph of f(x), we can start by identifying the key features of the sine function: y = sin(x).

The sine function oscillates between -1 and 1. It has a period of 2π and a y-intercept of 0. We can obtain the graph of y = sin(x) by plotting a few points and joining them with a smooth curve. Now, let's consider the function y = sin(x-45°). We can obtain this graph by translating the graph of y = sin(x) to the right by 45°. This means that the first peak of the sine function occurs at x = 45°, and the last peak occurs at x = 45° + 2π.

Finally, we add 2 to this function to get the graph of y = sin(x-45°) + 2. This translates the entire graph upwards by 2 units. Here's what it looks like: We can see that the graph of y = sin(x-45°) + 2 oscillates between 1 and 3.

This confirms that the range of the function is [1, 3].

Let's learn more about the sine function:

https://brainly.com/question/12595712

#SPJ11

Use the accompanying paired data consisting of weights of large cars (pounds) and highway fuel consumption (mi/gal). Let x represent the weight of a car and let y represent the highway fuel consumption. Use the given weight and the given confidence level to construct a prediction interval estimate of highway fuel consumption. Use x = 4200 pounds with a 99% confidence level. Click the icon to view the car weight and highway fuel consumption data. Find the indicated prediction interval. mi/gal

Answers

To construct a prediction interval estimate of highway fuel consumption for a car weighing 4200 pounds at a 99% confidence level, we need to use the given paired data and perform the necessary calculations.

1. Collect the paired data consisting of car weights and corresponding highway fuel consumption.

2. Calculate the sample mean and sample standard deviation of the highway fuel consumption.

3. Determine the critical value for a 99% confidence level. This critical value depends on the sample size and the desired confidence level.

4. Calculate the standard error of the estimate using the sample standard deviation and the square root of the sample size.

5. Use the critical value and the standard error to find the margin of error.

6. Calculate the lower and upper bounds of the prediction interval by subtracting and adding the margin of error to the sample mean, respectively.

Learn more about critical value  here: brainly.com/question/32058318

#SPJ11

Find the Laplace transform F(s) = L{f(t)} of the function f(t) = e²t-12 h(t-6), defined on the interval t > 0. F(s) = L {e²t-12 (t-6)} =

Answers

The Laplace transform of the function f(t) = e²t-12 h(t-6) is given by F(s) = L{e²t-12 (t-6)}. To compute the Laplace transform, we can apply the linearity property of the transform.

The Laplace transform of e²t is 1/(s-2), and the Laplace transform of h(t-6) is e^(-6s)/s.

Using the property of multiplication in the Laplace domain, we can multiply the individual Laplace transforms to obtain F(s) = 1/(s-2) * e^(-6s)/s.

Simplifying further, we can rewrite F(s) as (e^(-6s))/(s(s-2)).

Therefore, the Laplace transform of f(t) = e²t-12 h(t-6) is F(s) = (e^(-6s))/(s(s-2)).

Learn more about Laplace Transformation here: brainly.com/question/20463187


#SPJ11

The road adjacent to badminton court at Central
University, Lucknow, needed repair. So, the university
authorities hired Parikh to do the job. Parikh selected a
certain number of workers and assured the university
that work will be done in 10 days. Unfortunately, 4
workers were absent from the beginning and the task
took 50 days to complete. Can you tell us how many
workers Parikh hired initially.​

Answers

Parikh initially hired 5 workers to complete the job in 10 days.

Let's solve this problem using the concept of work rate.

Let's assume that Parikh initially hired "x" workers to complete the job in 10 days.

We can set up the equation as follows:

Work rate [tex]\times[/tex] Time = Total Work.

The work rate represents the amount of work done by each worker per day.

Since Parikh hired "x" workers, the work rate would be "x" times the work rate of one worker.

Now, let's consider the scenario where 4 workers were absent from the beginning.

This means that only (x - 4) workers were available to work.

The time taken to complete the task increased to 50 days.

We can set up another equation using the work rate:

(x - 4) [tex]\times[/tex] 50 = x [tex]\times[/tex] 10

This equation states that the work done by (x - 4) workers in 50 days should be equal to the work done by x workers in 10 days.

Let's solve this equation:

50x - 200 = 10x

Simplifying:

50x - 10x = 200

40x = 200

x = 200 / 40

x = 5

Therefore, Parikh initially hired 5 workers to complete the job in 10 days.

However, it's important to note that this solution assumes that the work rate remains constant throughout the project.

In reality, the work rate can vary due to various factors, such as fatigue or efficiency.

For similar question on work done.

https://brainly.com/question/8119756

#SPJ8

why can't a proper ideal of R contain a unit if R is a
ring with identity element 1?

Answers

A proper ideal of a ring R is a subset of R that is an ideal of R and does not contain the identity element 1. This is because if a proper ideal of R contains a unit, then it would also contain all the elements of R.

To understand why a proper ideal cannot contain a unit, let's consider the definition of an ideal. An ideal of a ring R is a subset I of R that satisfies two conditions: (1) for any x, y in I, their sum x + y is also in I, and (2) for any x in I and any r in R, the product rx and xr are both in I.

Now, if a proper ideal I contains a unit u (where u is an element of R and u ≠ 0), then by the second condition of the ideal definition, for any x in I, the product ux is also in I. But since u is a unit, there exists an element v in R such that uv = 1. Therefore, for any x in I, we have x = 1x = (uv)x = u(vx). Since vx is in R, it follows that x is in I. This means that the proper ideal I would actually be equal to the entire ring R, contradicting the assumption that I is a proper ideal.

Hence, a proper ideal of a ring with an identity element 1 cannot contain a unit.

Learn more about identity elements here: brainly.com/question/366935
#SPJ11

solve in 50 mins i will thumb up my candidate number 461 if needed anywhere (b Amli: You are driving on the forest roads of Amli, and the average number of potholes in the road pcr kilometer equals your candidate number on this exam. i. Which process do you need to use to do statistics about the potholes in the Amli forest roads,and what are the values of the parameters for this process? ii. What is the probability distribution of the number of potholes in the road for the next 100 meters? iii. What is the probability that you will find more than 30 holes in the next 100 meters?

Answers

i. In order to do statistics about the potholes in the Amli forest roads, the Poisson process can be used. The values of the parameters for this process are given below:

Parameter λ: The average number of potholes per kilometer.

The interval between two potholes is exponentially distributed.

ii. Probability distribution of the number of potholes in the road for the next 100 meters: Poisson distribution is used to calculate the probability of the number of potholes in the road for the next 100 meters. The mean value of λ in a hundred meters is 100/1000 * 461 = 46.1 λ=46.1

iii. Probability that you will find more than 30 holes in the next 100 meters: Probability that you will find more than 30 holes in the next 100 meters can be calculated as follows:

P(X>30) = 1 - P(X≤30)P(X>30) = 1 - ΣP(X=k) from k=0 to k=30

P(X=k) = λ^k * e^-λ/k!P(X>30) = 1 - [P(X=0) + P(X=1) + P(X=2) + ... + P(X=30)]P(X>30)

= 1 - [e^-λ(λ^0/0! + λ^1/1! + λ^2/2! + ... + λ^30/30!)]P(X>30)

= 1 - [e^-46.1(1 + 46.1/1! + 1060.21/2! + ... + 7.77 x 10^21/30!)]

Therefore, the probability that you will find more than 30 holes in the next 100 meters is 0.154 or approximately 15.4%.

Learn more about Probability: https://brainly.com/question/31828911

#SPJ11

If two states are selected at random from a group of 30 states, determine the number of possible outcomes if the group of states are selected with replacement or without replacement. If the states are selected with replacement, there are possible outcomes If the states are selected without replacement, there are possible outcomes
Previous question
Next question

Answers

If two states are selected at random from a group of 30 states, the number of possible outcomes if the group of states is selected with replacement or without replacement can be calculated as follows: With Replacement: If the states are selected with replacement, then the total number of possible outcomes is equal to the product of the number of states in the group and the number of states that can be selected again.

The total number of states in the group is 30, and since there are no restrictions on selecting a state again, the number of possible outcomes is given by:30 x 30 = 900. Total possible outcomes with replacement = 900Without Replacement:  If the states are selected without replacement, the total number of possible outcomes is given by the product of the number of states in the group and the number of states that can be selected next. The first state can be selected from the group of 30 states, and once it has been selected, the second state can be selected from the remaining 29 states. Therefore, the total number of possible outcomes is given by:30 x 29 = 870Total possible outcomes without replacement = 870Therefore, if two states are selected at random from a group of 30 states, the number of possible outcomes if the group of states is selected with replacement or without replacement are 900 and 870, respectively.

If the states are selected with replacement, there are 900 possible outcomes, and if the states are selected without replacement, there are 435 possible outcomes.

If the states are selected with replacement, there are 900 possible outcomes. This is because for each selection, there are 30 options, and since there are two selections, the total number of outcomes is 30 * 30 = 900.

If the states are selected without replacement, there are 435 possible outcomes. In this case, for the first selection, there are 30 options, but for the second selection, there are only 29 remaining options. Therefore, the total number of outcomes is 30 * 29 = 870.

To know more about outcomes,

https://brainly.com/question/23487927

#SPJ11


#3
Use a graphing calculator to solve the equation. Round your answer to two decimal places. ex=x²-1 O (2.54 O (-1.15) O 1-0.71) O (0)

Answers

The solution to the equation is x = -1.00 and x = 1.00.To summarize, the solution to the equation x²-1 using a graphing calculator is

x = -1.00 and x = 1.00.

Given equation is x²-1.To solve the equation using a graphing calculator, follow the steps below.Step 1: Enter the equation into the calculator. Press the "y=" key on the calculator and enter the equation. In this case, it is x²-1. Step 2: Graph the equation.Press the "graph" button on the calculator to graph the equation. Step 3: Find the x-intercepts. Look at the graph and find where the graph intersects the x-axis.

These points are called the x-intercepts. In this case, the x-intercepts are at approximately -1 and 1. Step 4: Round the answer.Rounding the answer to two decimal places gives -1.00 and 1.00. Therefore, the solution to the equation is

x = -1.00 and x = 1.00.

To summarize, the solution to the equation x²-1 using a graphing calculator is

x = -1.00 and x = 1.00.

To know more about equation  visit:-

https://brainly.com/question/29657983?

#SPJ11

I need solution for following problem

Make a solution that tests the probability of a certain score when rolling x dice. The user should be able to choose to roll eg 4 dice and test the probability of a selected score eg 11. The user should then do a number of simulations and answer how big the probability is for the selected score with as many dice selected. There must be error checks so that you cannot enter incorrect sums, for example, it is not possible to get the sum 3 if you roll 4 dice.

How many dices do you want to throw? 4

Which number do you want the probability for? 11

The probability the get the number 11 with 4 dices is 7.91%.

Answers

To calculate the probability of obtaining a specific sum when rolling multiple dice, you can use the formula  [tex]P(S) = (F / T) * 100[/tex].

P(S) is the probability of obtaining the desired sum.

F is the number of favorable outcomes (combinations resulting in the desired sum).

T is the total number of possible outcomes.

In this case, you can substitute the values into the formula to find the probability. Let's say you want to calculate the probability of getting a sum of 11 with 4 dice:

F = number of combinations resulting in a sum of 11

T = total number of possible combinations ([tex]6^4[/tex], as each die has 6 possible outcomes)

Then, the formula becomes:

P(11) = (F / T) * 100

By calculating the ratio of favorable outcomes to total outcomes and multiplying it by 100, you will obtain the probability as a percentage.

Please note that to determine the number of favorable outcomes, you may need to consider all possible combinations and count the ones that result in the desired sum.

Learn more about probability here:

https://brainly.com/question/32117953

#SPJ11

Determine the volume generated of the area bounded by y=√x and y= ½ x rotated around the y-axis.
a. (64/5)π
b. (8/15)π
c. (128/25)π
d. (64/15)

Answers

To determine the volume generated by rotating the area bounded by the curves y = √x and y = ½x around the y-axis, we can use the method of cylindrical shells. By setting up the integral and evaluating it, we find that the volume is equal to (64/15)π.

To find the volume, we use the method of cylindrical shells, which involves integrating the circumference of the shells multiplied by their heights. In this case, the height of each shell is the difference between the y-values of the two curves: (√x - ½x).

We integrate with respect to x from the lower bound to the upper bound, which are the x-values where the two curves intersect: x = 0 and x = 4.

Setting up the integral and evaluating it, we find that the volume is equal to ∫(0 to 4) 2πx(√x - ½x) dx. This simplifies to (64/15)π, which is the final answer.

Therefore, the volume generated by rotating the area bounded by the curves y = √x and y = ½x around the y-axis is (64/15)π.

To learn more about integral, click here:

brainly.com/question/31744185

#SPJ11

A flagpole and a building stand on the same horizontal level.From the point p at the bottom of the building, the angle of elevation ot the top t of the flagpole is 65° .from the top of building the angleof elevation of the point t is 25.if the building is 20°high calculate the:

Distance pt,height of the flagpole

Distance qt

Answers

From point P to T (pt): pt = 20 / tan(65°) ≈ 11.07 units.

Height of flagpole cannot be determined without knowing its value.

The distance from point P to point T (pt) can be calculated using trigonometry. Given that the angle of elevation from point P to point T is 65° and the height of the building is 20 units, we can set up the following equation:

tan(65°) = height of flagpole / pt

Solving for pt, we get:

pt = height of flagpole / tan(65°)

Substituting the given height of the building (20 units), we have:

pt = 20 / tan(65°)

Calculating this value, we find that pt is approximately 11.07 units.

To find the height of the flagpole, we can use the angle of elevation from the top of the building (point T) to point Q. Given that this angle is 25°, we can set up the following equation:

tan(25°) = height of flagpole / qt

Rearranging the equation, we find:

qt = height of flagpole / tan(25°)

Since we don't know the height of the flagpole yet, we can substitute it with a variable h:

qt = h / tan(25°)

Hence, we cannot calculate the exact value of qt without knowing the height of the flagpole (h).

For more such questions on Height, click on:

https://brainly.com/question/28122539

#SPJ8

Which ONE of the following is NOT the critical point of the function f(x,y)=xye-(x² + y²)/2?
A. None of the choices in this list.
B. (0,0).
C. (1,1).
D. (-1,-1).
E. (0.1).

Answers

The critical point of the function f(x,y) = xy*e^(-(x^2 + y^2)/2) is (0,0). The critical points of a function occur where the gradient is zero or undefined.

To find the critical points of f(x,y), we need to calculate the partial derivatives with respect to x and y and set them equal to zero.

Let's find the partial derivatives:

∂f/∂x = ye^(-(x^2 + y^2)/2) - xy^2e^(-(x^2 + y^2)/2)

∂f/∂y = xe^(-(x^2 + y^2)/2) - xy^2e^(-(x^2 + y^2)/2)

Setting both partial derivatives to zero, we have:

ye^(-(x^2 + y^2)/2) - xy^2e^(-(x^2 + y^2)/2) = 0     ...(1)

xe^(-(x^2 + y^2)/2) - xy^2e^(-(x^2 + y^2)/2) = 0     ...(2)

From equation (2), we can simplify it as:

x = xy^2                  ...(3)

Plugging this into equation (1), we get:

ye^(-(x^2 + y^2)/2) - (xy^2)^2e^(-(x^2 + y^2)/2) = 0

ye^(-(x^2 + y^2)/2) - x^2y^4e^(-(x^2 + y^2)/2) = 0

Factoring out ye^(-(x^2 + y^2)/2), we have:

ye^(-(x^2 + y^2)/2)(1 - xy^2e^(-(x^2 + y^2)/2)) = 0

This equation holds true if either ye^(-(x^2 + y^2)/2) = 0 or 1 - xy^2e^(-(x^2 + y^2)/2) = 0.

The first equation, ye^(-(x^2 + y^2)/2) = 0, implies y = 0.

The second equation, 1 - xy^2e^(-(x^2 + y^2)/2) = 0, implies x = 0 or y = ±1.

Considering these results, we can see that the only critical point that satisfies both equations is (0,0). Therefore, (0,0) is the critical point of the function f(x,y)=xye^(-(x^2 + y^2)/2).

To know more about critical points, refer here:

https://brainly.com/question/31308189#

#SPJ11

There are two pockets X and Y. There are five cards in each pocket. A number is written on each card. The numbers written on the cards in pocket X are "2", "3", "4", "5" and "5". The numbers written on the cards in pocket Y are "4", "5", "6", "-1" and "-1". We randomly select a card from each pocket. X denotes the number written on the card selected from pocket X. Y denotes the number written on the card selected from pocket Y. X and Y are independent. The expected value of X, namely E[X], is [...]

Answers

The expected value of X, denoting the number written on the card selected from pocket X, can be calculated by taking the average of the numbers on the cards in pocket X.

To calculate the expected value of X, we need to find the average value of the numbers written on the cards in pocket X. The numbers in pocket X are 2, 3, 4, 5, and 5. By summing up these numbers (2 + 3 + 4 + 5 + 5) and dividing the sum by the total number of cards in pocket X (5), we obtain the expected value of X.

(2 + 3 + 4 + 5 + 5) / 5 = 19 / 5 = 3.8

Therefore, the expected value of X, denoting the number written on the card selected from pocket X, is 3.8.

The concept of expected value is a way to determine the average value we can expect from a random variable. In this case, since the selection of a card from pocket X is independent of the selection from pocket Y, the expected value of X can be calculated solely based on the numbers in pocket X. It represents the long-term average value we would expect to obtain if we were to repeat this random selection process many times.

Learn more about expected value here:

brainly.com/question/13749480

#SPJ11

1. Find the Laplace transform of f(t)=e3t

using the definition of the Laplace transform.

2. Find L{f(t)}

.

a. f(t)=3t2−5t+7

b. f(t)=2e−4t

c. f(t)=3 cos 2t−sin 5t

d. f(t)=te2t

e. f(t)=e−tsin 3t

Answers

The Laplace transform of f(t)=e3t is given by L{f(t)} = 1/(s-3). The Laplace transforms of f(t)=3t2−5t+7, f(t)=2e−4t, f(t)=3 cos 2t−sin 5t, f(t)=te2t, and f(t)=e−tsin 3t are given by L{f(t)} = (3s^3-15s^2+42s+7)/(s^3), L{f(t)} = 2/(s+4), L{f(t)} = (6)/(s^2+4)-(5)/(s^2+25), L{f(t)} = (2e^2)/((s-2)^2), and L{f(t)} = 3/((s+1)^2+9), respectively.ms:

1. Find the Laplace transform of f(t)=e3t using the definition of the Laplace transform.

The Laplace transform of f(t)=e3t is given by:

L{f(t)} = \int_0^\infty e^{-st}e^{3t}dt = \frac{1}{s-3}

2. Find L{f(t)} for the following functions

a. f(t)=3t2−5t+7

L{f(t)} = \int_0^\infty e^{-st}(3t^2-5t+7)dt = \frac{3s^3-15s^2+42s+7}{s^3}

b. f(t)=2e−4t

L{f(t)} = \int_0^\infty e^{-st}(2e^{-4t})dt = \frac{2}{s+4}

c. f(t)=3 cos 2t−sin 5t

L{f(t)} = \int_0^\infty e^{-st}(3 cos 2t−sin 5t)dt = \frac{6}{s^2+4}-\frac{5}{s^2+25}

d. f(t)=te2t

L{f(t)} = \int_0^\infty e^{-st}(te^{2t})dt = \frac{2e^2}{(s-2)^2}

e. f(t)=e−tsin 3t

L{f(t)} = \int_0^\infty e^{-st}(e^{-t}sin 3t)dt = \frac{3}{(s+1)^2+9}

Learn more about Laplace transform here:

brainly.com/question/30759963

#SPJ11

Consider the following functions.
f(x) = 8 / (x-4) and g(x) = 2x - 6 (a) Find the domain of f(x). (Enter your answer using interval notation.) ____
(b) Find the domain of g(x). (Enter your answer using interval notation.)
____
(c) Find (fog)(x). (Simplify your answer completely.)
(fog)(x) = ____ (d) Find the domain of (fog)(x). (Enter your answer using interval notation.)
_____

Answers

Given functions are:[tex]$f(x) = \frac{8}{x - 4}$[/tex] and [tex]g(x) = 2x - 6[/tex]. Now we have to find out the domain of the given functions and also find out the domain of f(g(x)) which is (fog)(x).

(a) Domain of f(x)Domain of f(x) is the set of all the real numbers except the number 4.

Because at x = 4, the denominator of the function f(x) becomes zero, which means the function is undefined at x = 4.

Domain of [tex]f(x) = (-∞, 4) U (4, +∞)[/tex]

(b) Domain of g(x) Domain of g(x) is the set of all the real numbers because the domain of a linear function is all the real numbers

.Domain of[tex]g(x) = (-∞, +∞)(c) (fog)(x)[/tex]

To find (fog)(x),

we need to substitute g(x) into the function f(x).

[tex]fog(x) = f(g(x))fog(x)[/tex]

[tex]= f(2x - 6)[/tex]

Replace the g(x) in [tex]f(x) with 2x - 6.fog(x)[/tex]

[tex]=\frac{8}{(2x - 6 - 4)fog(x)}\\=\frac{8}{2(x - 5)fog(x)}\\=\frac{4}{(x - 5)}[/tex]

Therefore, [tex](fog)(x)=\frac{4}{(x - 5)}[/tex]

(d) Domain of (fog)(x)The domain of (fog)(x) is the same as the domain of g(x) which is all the real numbers except when the denominator is zero, so the function is undefined.

In this case, the denominator can never be zero, so the domain of (fog)(x) is all the real numbers.

Domain of[tex](fog)(x) = (-∞, +∞)[/tex]

Answer:(a) Domain of [tex]f(x) = (-∞, 4) U (4, +∞)[/tex]

(b) Domain of [tex]g(x) = (-∞, +∞)[/tex]

(c) [tex](fog)(x)=\frac{4}{(x - 5)}[/tex]

(d) Domain of [tex](fog)(x) = (-∞, +∞)[/tex]

To know more about real numbers visit:

https://brainly.com/question/9876116

#SPJ11

Consider two friends Alfred (A) and Bart (B) with identical income IĄ = IB = 100, they both like only two goods (x₁ and x₂). That are currently sold at prices p₁ = 1 and p2 = 4. The only difference between them are preferences, in particular, Alfred preferences are represented by the utility function:
uA (x1, x2) = x1 0.5 x2 0.5
while Bart's preferences are represented by:
UB(x₁, x₂) = min{x₁,4x2}
1. Do the the following:
a) Define and draw the budget constraint for each consumer.
b) Determine the Marshallian demand curve (as a function of income and prices for each good for Alfred and Bart. What quantities are going to be consumed?
c) Tror False Consumers with different preferences always Loice different bundles
d) Can you determine who is better by comparing utility?

Answers

The budget constraint for Alfred can be represented by the equation: p₁x₁ + p₂x₂ = I, where p₁ = 1, p₂ = 4, and I = 100. For Bart, the budget constraint is given by: p₁x₁ + p₂x₂ = I, with the same values for prices and income.

The Marshallian demand curve represents the quantity of each good that Alfred and Bart will consume at different price levels. To find this, we need to solve the budget constraint equation for each good.

For Alfred:

p₁x₁ + p₂x₂ = I

1x₁ + 4x₂ = 100

x₁ = 100 - 4x₂

For Bart:

p₁x₁ + p₂x₂ = I

1x₁ + 4x₂ = 100

x₁ = 100 - 4x₂

Substituting the values of x₁ into the utility functions, we can find the quantities consumed:

For Alfred:

uA(x₁, x₂) = x₁^0.5 * x₂^0.5

uA(100 - 4x₂, x₂) = (100 - 4x₂)^0.5 * x₂^0.5

For Bart:

uB(x₁, x₂) = min{x₁, 4x₂}

uB(100 - 4x₂, x₂) = min{100 - 4x₂, 4x₂}

True, consumers with different preferences will generally choose different bundles of goods due to their varying utility functions and budget constraints.

d) We cannot determine who is better by comparing utility alone, as utility is subjective and varies from person to person. The utility functions of Alfred and Bart represent their individual preferences, and what might be preferred by one person may not be the same for another. Utility is a personal measure and cannot be compared across individuals.

Learn more about budget constraints

brainly.com/question/31077531

#SPJ11

(i) In R³, let M be the span of v₁ = (1,0,0) and v2 = (1, 1, 1). Find a nonzero vector v3 in Mt. Apply Gram-Schmidt process on {V1, V2, V3}. (ii) Suppose V is a complex finite dimensional IPS. If T is a linear trans- formation on V such that (T(x), x) = 0 for all x € V, show that T = 0. (Hint: In (T(x), x) = 0, replace x by x+iy and x-iy.)

Answers

The vector v3 is a nonzero vector in M, which can be found using the Gram-Schmidt process. The operator T is a zero operator, which can be shown using the fact that (T(x), x) = 0 for all x in V.

(i) The vector v3 = (-1, 1, 1) is a nonzero vector in M. To find this vector, we can use the Gram-Schmidt process on the vectors v1 and v2. The Gram-Schmidt process works by first finding the projection of v2 onto v1. This projection is given by

proj_v1(v2) = (v2 ⋅ v1) / ||v1||^2 * v1

In this case, we have

proj_v1(v2) = ((1, 1, 1) ⋅ (1, 0, 0)) / ||(1, 0, 0)||^2 * (1, 0, 0) = (1/2) * (1, 0, 0) = (1/2, 0, 0)

We then subtract this projection from v2 to get the vector v3. This gives us

v3 = v2 - proj_v1(v2) = (1, 1, 1) - (1/2, 0, 0) = (-1, 1, 1)

(ii) To show that T = 0, we can use the fact that (T(x), x) = 0 for all x in V. We can then replace x by x + iy and x - iy to get

(T(x + iy), x + iy) = 0 and (T(x - iy), x - iy) = 0

Adding these two equations, we get

(T(x + iy) + T(x - iy), x + iy - (x - iy)) = 0

This simplifies to

(2iT(x), 2ix) = 0

Since this equation holds for all x in V, we must have 2iT(x) = 0 for all x in V. This implies that T(x) = 0 for all x in V, so T = 0.

Learn more about Gram-Schmidt process here:

brainly.com/question/30761089

#SPJ11

the curve that passes through the point (1 1) and whose slope at any point xy is equal to 3y x

Answers

The equation of curve that passes through the point (1, 1) and whose slope at any point xy is equal to 3y x is:y = [(3 + e^(4√3)) / (2e^(2√3))]e^(√(9x² + 3)x) + [(3 - e^(4√3)) / (2e^(-2√3))]e^(-√(9x² + 3)x).

Let us consider a curve that passes through the point (1, 1) and whose slope at any point xy is equal to 3yx. Let the curve be defined by the function y = f (x). Now we want to find the equation of this curve.

To do so, we will use the method of separable variables. We have:y' = 3yx

Differentiating both sides with respect to x, we obtain:y'' = 3y + 3xy' = 3y + 3x(3yx) = 3y + 9x²y

Simplifying this equation, we obtain:y'' - 3y = 9x²yNow we can use the characteristic equation method to find the general solution of this differential equation.

Let y = e^rx. Then:y' = re^rx and y'' = r²e^rx

Substituting these expressions into the differential equation, we get:r²e^rx - 3e^rx = 9x²e^rxSimplifying this equation, we obtain:r² - 3 = 9x²or:r² = 9x² + 3or:r = ±√(9x² + 3)

Therefore, the general solution of the differential equation is:y = c₁e^(√(9x² + 3)x) + c₂e^(-√(9x² + 3)x)where c₁ and c₂ are constants to be determined by the initial condition (1, 1).

Now we use the initial condition to find the values of c₁ and c₂.

We have:y(1) = c₁e^(√(9+3)) + c₂e^(-√(9+3))= c₁e^(2√3) + c₂e^(-2√3) = 1Also, we can write:y'(x) = 3yx(x), so y'(1) = 3y(1) = 3(c₁e^(2√3) + c₂e^(-2√3)) = 3.

Substituting the second equation into the first, we obtain:c₁e^(2√3) + c₂e^(-2√3) = 1/ (c₁e^(2√3) + c₂e^(-2√3)) × 3= 3/ (c₁e^(2√3) + c₂e^(-2√3))

Multiplying both sides by (c₁e^(2√3) + c₂e^(-2√3)), we get: c₁e^(2√3) + c₂e^(-2√3) = 3

Solving this system of equations for c₁ and c₂, we obtain:c₁ = (3 + e^(4√3)) / (2e^(2√3)), c₂ = (3 - e^(4√3)) / (2e^(-2√3))

Know more about the method of separable variables

https://brainly.com/question/30481497

#SPJ11

Inflation is causing prices to rise according to the exponential growth model with a growth rate of 3.2%. For the item that costs $540 in 2017, what will be the price in 2018?

Answers

According to the exponential growth model, the item should cost about $556.64 in 2018 at a growth rate of 3.2%.

Formula: P(t) = P(0) * e^(r*t)

Where:

P(t) is the price at time t

P(0) is the initial price (at t=0)

r is the growth rate (expressed as a decimal)

t is the time elapsed (in years)

In this case, the initial price (P(0)) is $540, the growth rate (r) is 3.2% (or 0.032 as a decimal), and we want to find the price in 2018, which is one year after 2017 (t=1).

Substituting the given values into the formula, we have:

P(1) = $540 * e^(0.032 * 1)

Using a calculator or software, we can calculate the exponential term e^(0.032) 1.032470.

P(1) = $540 * 1.032470 $556.64

Therefore, based on the exponential growth model with a growth rate of 3.2%, the estimated price of the item in 2018 would be approximately $556.64.

To know more about growth model refer here:

https://brainly.com/question/30500128#

#SPJ11

3. Graph the region bounded by the functions y = x² and y = x + 2, set up and evaluate the integral that will give the area.

Answers

We evaluate the integral A = ∫[-1, 2] ((x + 2) - x²) dx to find the area of the region bounded by the given functions.

To graph the region bounded by y = x² and y = x + 2, we plot both functions on the same coordinate system. The region is the area between these two curves.

To find the area, we need to set up an integral that represents the difference in the y-values of the upper and lower functions as we integrate over the appropriate range of x-values.

The integral for calculating the area is given by A = ∫[a, b] (f(x) - g(x)) dx, where f(x) represents the upper function (in this case, y = x + 2), g(x) represents the lower function (y = x²), and [a, b] represents the x-values where the two functions intersect.

To evaluate the integral, we need to find the x-values where the two functions intersect. Setting x + 2 = x² and solving for x, we get x = -1 and x = 2 as the intersection points.

Finally, we evaluate the integral A = ∫[-1, 2] ((x + 2) - x²) dx to find the area of the region bounded by the given functions.

To learn more about functions click here, brainly.com/question/31062578

#SPJ11

determine if the following functions t : double-struck r2 → double-struck r2 are one-to-one and/or onto. (select all that apply.) (a) t(x, y) = (4x, y) one-to-one onto neither.
(a) T(x, y)-(2x, y) one-to-one onto U neither (b) T(x, y) -(x4, y) one-to-one onto neither one-to-one onto U neither (d) T(x, y) = (sin(x), cos(y)) one-to-one onto U neither

Answers

T(x, y) = (4x, y) is onto, T(x, y) = (x^4, y) is one-to-one but not onto, T(x, y) = (sin(x), cos(y)) is neither one-to-one nor onto.

(a) The function t(x, y) = (4x, y) is not one-to-one because for any y, there are infinitely many x values that map to the same (4x, y).

For example, t(1, 0) = t(0.25, 0) = (4, 0), which means different input pairs map to the same output pair.

However, the function is onto because for any (a, b) in ℝ², we can choose x = a/4 and y = b, and we have t(x, y) = (4x, y) = (a, b).

(b) The function T(x, y) = (x^4, y) is one-to-one because different input pairs result in different output pairs.

If (x₁, y₁) ≠ (x₂, y₂), then T(x₁, y₁) = (x₁^4, y₁) ≠ (x₂^4, y₂) = T(x₂, y₂).

However, the function is not onto because not every point in ℝ² is mapped to by T.

For example, there is no input (x, y) such that T(x, y) = (-1, 0).

(c) The function T(x, y) = (sin(x), cos(y)) is not one-to-one because different input pairs can result in the same output pair.

For example, T(0, 0) = T(2π, 0) = (0, 1).

Additionally, the function is not onto because not every point in ℝ² is mapped to by T.

For example, there is no input (x, y) such that T(x, y) = (2, 2).

To know more about onto refer here:

https://brainly.com/question/31489686#

#SPJ11

Use the limit definition to find the derivative of the function.
f(x) = 3x² - 3x f(x +h)-f(x)
First, find f(x+h) – f(x)
Next, simplify the numerator.
Divide out the h.
So now, find the limit
Limh→[infinity] f(x+h- f(x) / h +___________

Answers

Dividing this expression by h and taking the limit as h approaches 0, we found the derivative to be 6x - 3. Limh→[infinity] f(x+h- f(x) / h + 6x - 3.

To find the derivative of the function f(x) = 3x² - 3x using the limit definition, we start by finding the expression f(x + h) - f(x), where h represents a small change in x.

f(x + h) = 3(x + h)² - 3(x + h) = 3(x² + 2xh + h²) - 3x - 3h

Now, we can subtract f(x) = 3x² - 3x from f(x + h):

f(x + h) - f(x) = [3(x² + 2xh + h²) - 3x - 3h] - [3x² - 3x]

Simplifying the numerator:

f(x + h) - f(x) = 3x² + 6xh + 3h² - 3x - 3h - 3x² + 3x

The terms 3x² and -3x² cancel out, as well as 3x and -3x:

f(x + h) - f(x) = 6xh + 3h² - 3h

Now, we can divide this expression by h to find the difference quotient:

[f(x + h) - f(x)] / h = (6xh + 3h² - 3h) / h

Simplifying further:

[f(x + h) - f(x)] / h = 6x + 3h - 3

Finally, we take the limit as h approaches 0:

lim(h→0) [f(x + h) - f(x)] / h = lim(h→0) (6x + 3h - 3)

The limit of this expression is simply 6x - 3.

Therefore, the derivative of f(x) = 3x² - 3x is f'(x) = 6x - 3.

In summary, we used the limit definition of the derivative to find the derivative of the function f(x) = 3x² - 3x.

By calculating the expression f(x + h) - f(x) and simplifying, we obtained (6xh + 3h² - 3h) / h. Dividing this expression by h and taking the limit as h approaches 0, we found the derivative to be 6x - 3.

To know more about derivative click here

brainly.com/question/29096174

#SPJ11

Find the expressions all valves below.
i) (1+i)^5/7
ii) 1^(1-i)

Answers

i) The expression (1+i)^(5/7) can be written in polar form as (2^(1/2) * e^(iπ/4))^(5/7). Using De Moivre's theorem, we can simplify this expression to 2^(5/14) * e^(i(5π/28)).

ii) The expression 1^(1-i) simplifies to 1.

i) To find the expression of (1+i)^(5/7), we can represent (1+i) in polar form. The magnitude of (1+i) is √2, and the argument is π/4. Therefore, we have (1+i) = √2 * e^(iπ/4).

Using De Moivre's theorem, which states that (r * e^(iθ))^n = r^n * e^(iθn), we can simplify the expression. In this case, r = √2, θ = π/4, and n = 5/7.

Applying De Moivre's theorem, we get (1+i)^(5/7) = (√2 * e^(iπ/4))^(5/7) = 2^(5/14) * e^(i(5π/28)). Therefore, the expression simplifies to 2^(5/14) * e^(i(5π/28)).

ii) The expression 1^(1-i) simplifies to 1 raised to the power of (1-i). Any non-zero number raised to the power of 0 is equal to 1. Since 1 is a non-zero number, we have 1^(1-i) = 1.

Therefore, the expressions are:

i) (1+i)^(5/7) = 2^(5/14) * e^(i(5π/28)).

ii) 1^(1-i) = 1.

To learn more about De Moivre's theorem click here: brainly.com/question/29750103


#SPJ11



9. [1/5 Points]
DETAILS
PREVIOUS ANSWERS
TANFIN12 1.3.014.
A manufacturer has a monthly fixed cost of $57,500 and a production cost of $9 for each unit produced. The product sells for $14/unit. (a) What is the cost function?
C(x)
7500+9xx
(b) What is the revenue function? R(x) = 14x
(c) What is the profit function?
P(x) = 5x – 7500 | x
(d) Compute the profit (loss) corresponding to production levels of 9,000 and 14,000 units.
P(9,000) 37500
P(14,000)
=
62500
X
Need Help?
Read It
MY

Answers

(a) The cost function C(x) represents the total cost associated with producing x units. In this case, the monthly fixed cost is $57,500, and the production cost per unit is $9. The cost function can be expressed as:

[tex]C(x) &= \text{Fixed cost} + (\text{Variable cost per unit} \times \text{Number of units}) \\C(x) &= \$57,500 + (\$9 \times x)[/tex]

(b) The revenue function R(x) represents the total revenue generated from selling x units. The selling price per unit is $14, so the revenue function is simply:

[tex]\[R(x) &= \text{Selling price per unit} \times \text{Number of units} \\R(x) &= \$14 \times x\][/tex]

(c) The profit function P(x) represents the total profit (or loss) obtained from producing and selling x units. It is calculated by subtracting the total cost from the total revenue:

[tex]P(x) &= R(x) - C(x) \\P(x) &= (\$14 \cdot x) - (\$57,500 + (\$9 \cdot x)) \\P(x) &= \$14x - \$57,500 - \$9x \\P(x) &= \$5x - \$57,500[/tex]

(d) To compute the profit (or loss) corresponding to production levels of 9,000 and 14,000 units, we substitute the values of x into the profit function:

[tex]\[P(9,000) &= \$5 \times 9,000 - \$57,500 \\P(9,000) &= \$45,000 - \$57,500 \\P(9,000) &= -\$12,500 \quad (\text{loss}) \\\\P(14,000) &= \$5 \times 14,000 - \$57,500 \\P(14,000) &= \$70,000 - \$57,500 \\P(14,000) &= \$12,500 \quad (\text{profit})\][/tex]

Therefore, at a production level of 9,000 units, the company incurs a loss of $12,500, while at a production level of 14,000 units, the company earns a profit of $12,500.

To know more about Selling Price visit-

brainly.com/question/27796445

#SPJ11


Discrete mathematics question, pls answer :
Question 6. Construct the truth table and then derive the Principal Conjunctive Normal Form(CNF) for (p¬q) → r. Please scan and upload your answer as a separate file.

Answers

Given that the logical statement is (p ¬q) → r.

The first step is to construct the truth table as follows: p q r p ¬q (p ¬q) → r T T T F T F T T F F T T T F T F F T T T F T F

The next step is to derive the principal conjunctive normal form (CNF) for the given logical statement. From the truth table, the values that give true as the result are:(p ¬q) → r = T From the CNF, all the conjuncts must be true. So, the CNF of (p ¬q) → r can be derived by the following steps:1. All the rows of the truth table where the value is T must be identified.2. In each of these rows, identify all the propositions (p, q, r) and their negations (¬p, ¬q, ¬r) that are true.3. Create a clause from each of these rows by combining the propositions with OR and placing them within brackets.4. Finally, combine the clauses with AND. Each clause represents a disjunction of literals (a variable or its negation). So, the CNF for (p ¬q) → r is: (p ∨ r) ∧ (q ∨ r) ∧ (¬p ∨ ¬q ∨ r)

Learn more about truth table:

https://brainly.com/question/28032966

#SPJ11

For (K, L) = 12K1/3L1/2 - 4K – 1, where K > 0,1 20, L TT = find the profit-maximizing level of K. Answer:

Answers

K2/3 = 12Hence, K = (12)3/2 = 20.784 Profit maximizing value of K is 20.784.

Given the production function, (K, L) = 12K1/3L1/2 - 4K – 1, where K > 0,1 ≤ 20, L = π. We need to find the profit-maximizing level of K.

Profit maximization occurs where Marginal Revenue Product (MRP) is equal to the Marginal Factor Cost (MFC).To determine the optimal value of K, we will derive the expressions for MRP and MFC.

Marginal Revenue Product (MRP) is the additional revenue generated by employing an additional unit of input (labor) holding all other factors constant. MRP = ∂Q/∂L * MR where, ∂Q/∂L is the marginal physical product of labor (MPPL)MR is the marginal revenue earned from the sale of output.

MRP = (∂/∂L) (12K1/3L1/2) * MRLMPPL = 6K1/3L-1/2MR = P = 10Therefore, MRP = 6K1/3L1/2 * 10 = 60K1/3L1/2The Marginal Factor Cost (MFC) is the additional cost incurred due to the use of one additional unit of the input (labor) holding all other factors constant.

MFC = Wages = 5 Profit maximization occurs where MRP = MFC.60K1/3L1/2 = 5K1/3Multiplying both sides by K-1/3L-1/2, we get;60 = 5K2/3L-1Therefore,K2/3 = 12Hence, K = (12)3/2 = 20.784Profit maximizing value of K is 20.784.

For more such questions on Profit maximizing

https://brainly.com/question/28253283

#SPJ8

Consider the sets
A = {1, 3, 5, 7, 9, 11}, B = {1, 4, 9, 16, 25}, C= {3, 6, 9, 12, 15).
Verify that (A n B) U C = (A U C) n (B U C) and (A U B) n C = (A n C) U (B n C).

Answers

Both given set equalities are verified.

To verify the given set equalities, let's analyze each expression separately.

1. (A n B) U C = (A U C) n (B U C)

Left-hand side (LHS):

(A n B) U C = ({1, 9}) U {3, 6, 9, 12, 15} = {1, 3, 6, 9, 12, 15}

Right-hand side (RHS):

(A U C) n (B U C) = ({1, 3, 5, 7, 9, 11} U {3, 6, 9, 12, 15}) n ({1, 4, 9, 16, 25} U {3, 6, 9, 12, 15})

                  = {1, 3, 5, 6, 7, 9, 11, 12, 15} n {1, 3, 4, 6, 9, 12, 15, 16, 25}

                  = {1, 3, 6, 9, 12, 15}

Since the LHS and RHS have the same elements, (A n B) U C = (A U C) n (B U C) holds true.

2. (A U B) n C = (A n C) U (B n C)

Left-hand side (LHS):

(A U B) n C = ({1, 3, 5, 7, 9, 11} U {1, 4, 9, 16, 25}) n {3, 6, 9, 12, 15}

            = {1, 3, 4, 5, 7, 9, 11, 16, 25} n {3, 6, 9, 12, 15}

            = {3, 9}

Right-hand side (RHS):

(A n C) U (B n C) = ({1, 3, 5, 7, 9, 11} n {3, 6, 9, 12, 15}) U ({1, 4, 9, 16, 25} n {3, 6, 9, 12, 15})

                  = {3, 9} U ∅

                  = {3, 9}

Since the LHS and RHS have the same elements, (A U B) n C = (A n C) U (B n C) holds true.

To know more about set operations, click here: brainly.com/question/11067029

#SPJ11

Smart TVs Smart tvs have seen success in the united states market. during the 2nd quater of a recent year, 41% of tvs sold in the untied states were smart tvs. Choose three households. Find the probabilities.

Answers

The probability of choosing three households with different types of TVs is [tex]0.1439[/tex].

Since 41% of TVs sold in the US were smart TVs, we can assume that the probability of a household owning a smart TV is also 41%. The probability of choosing a household that owns a smart TV is 0.41 and the probability of choosing a household that doesn't own a smart TV is 0.59.

Thus, the probability of choosing three households with different types of TVs can be calculated as: 0.41 × 0.59 × 0.59 = 0.1439 (rounded to four decimal places)Therefore, the probability of choosing three households with different types of TVs is [tex]0.1439[/tex].

Learn more about probability here:

https://brainly.com/question/31828911

#SPJ11

need help please
Find the domain of the function. f(x)=√5x-45 The domain is (Type your answer in interval notation.)

Answers

So, the domain of the function f(x) = √(5x - 45) is x ≥ 9, which can be expressed in interval notation as [9, ∞).

To find the domain of the function f(x) = √(5x - 45), we need to determine the values of x for which the function is defined.

The square root function (√) is defined only for non-negative values. Therefore, the expression inside the square root (5x - 45) must be greater than or equal to 0:

5x - 45 ≥ 0

Solving for x, we have:

5x ≥ 45

x ≥ 9

The function is defined for all values of x greater than or equal to 9.

To know more about function,

https://brainly.com/question/28897617

#SPJ11

Other Questions
A patient needs 3 L of D5W with 20 meq of potassium chloride to infuse over one day (24 hours). The DF is 15 gtt/mL. What is the correct rate of flow in gtt/min? Round to the nearest whole number. (6 marks) Let (G, +) and (G, +) be two subgroups of (R,+) so that Z+ C G G. If : G G is a group isomorphism with o(1) = 1, show that p(n) = n for all n Z+. Hint: consider using mathematical induction Linear Combinations In Exercises 1-4, write each vector as a linear combination of the vectors in S (if possible). 1. S = {(2, 1, 3), (5, 0,4)} (a) z = (-1, -2, 2) (b) v = (8,-1,27) (d) u = (1, 1, 1) Systems of Polar Equations Name: Task: Design a system of polar equations that has only one solution on 0 8 S 2. Requirements: names (first, last middle, etc.) 1. One of your equations must be r A, where A is the number of letters COMBINED in all of your bigger numbers. 2. Accept that r= A is a crazy number, but it shouldn't make the problem any more difficult, just uses 3. The other equation needs to be a limaon or a cardioid. Solve the system using the three methods below: Graphical (provide a desmos link and describe the solution below) Tabular Equations Please conduct a SWOT analysis on the development of ruraltourism in Bangladesh . Marriott has $1.489 (million) worth of inventory and their COGS are $10.720 million. Their average holding cost per unit per year is $13.3. What is the average inventory cost per unit for Marriott? Instruction: Round your answer to the nearest 50.01 The average inventory cost per unit ....... QucauvIE The question. "What are the distinguishing characteristics of effective leaders?" sparked what approach to the study of leadership17+ pull The University of Michigan studies identified addi Solve the system with the addition method: 6x + 4y 5x 4y -1 1 = 2 Answer: (2,y) Preview : Preview y Enter your answers as integers or as reduced fraction(s) in the form A/B. 2) The one-year interest rates over the next five years are expected to be 5%, 6%,7%, 8% and 9%. Given this information, what are the interest rates on a two-year bond and a five-year bond? 4 marks Ex find a power series representation for the function f(t)=1/4 9t^2 2. Let z1=[1+i/ 2, 1-i/2] and Z = [i/2, -1/2](a) Show that {z,z) is an orthonormal set in C. (b) Write the vector z = [ 2+4i, -2i] as a linear Z combination of z, and z. Read the HR strategy case and complete the three questions below. Case study: HR Strategy: E-Commerce Comes to Starlight book You are the HR director for Starlight Books in Cambridge, Ontario, a family-owned business since 1910. It has been very successful over the years, providing a wide range of books to its customers. Starlight Books motto is "You want it, we got - or we can get it fast." The company prides itself on high-quality customer service and the ability to live up to its motto fully. Starlight operations have spread from its single store in Toronto to thirty stores in Vancouver, Quebec, Nova Scotia, and Alberta. The company locates its stores in medium-sized towns because it found competition in the larger cities was too great. It has a highly efficient distribution system to ensure that all stores can live up to its motto. Starlight's workforce is hired from the local store areas. Stores provide part-time work for local high school students, and full-time employees tend to range in age from the mid-twenties to the late thirties. The company offers a profit-sharing scheme for all its full-time employees. Turnover among these staff members is relatively low, with most staying with the company for several years. Over the past couple of years, the company's profits have been dropping. Market research has indicated that much of the company's loss in revenue can be attributed to booksellers operating on the internet. After many soul searching, the current CEO of the company has decided to take the company online. Initially, the company will run all of its e-commerce operations from its home site in Cambridge and use its existing distribution center. As to what this move to e-commerce means for the company's existing storefront operations, the CEO will only say, "well, we'll just have to wait and see about that." Questions: 1. Outline the company's strengths and weaknesses related to this change in mode of operation [5 marks] 2. Outline the opportunities and threats associated with this change in business strategy. [5 marks] 3. Outline the HR practice implications for staff of such a change in operations, specifically a. What changes, if any, would be needed in how people are hired and what kinds of skills and qualifications they would need to have? [5 marks] b. How would you appraise the performance of staff in the new e-commerce operations, and how would that differ from staff in the store sites? [5 marks] c. What new training and development programs would be required? [5 marks] solving logrithmic equationplease provide step-by -steps thank youSolve for a. Simplify your answer. Do not use decimals. -7+log (x - 2) = -5 x = 12. In a classroom there are 30 students, 20 boys and 10 girls. Four students are selected to form a committee representing the class. Calculate the probability that the first two selected are boys and the next two girls; What is the probability that the committee has two girls and two boys? What is the probability that the first student selected is a boy? And the third? 13. Consider a computer system that generates randomly a key-word for a new user com- posed of 5 letters (eventually repeated) of an alphabet of 26 letters (no distinction is made between capital and lower case letters). Calculate the probability that there is no repeated letters in the key-word. d) What does it mean to be "98% confident" in this problem? 98% of all times will fall within this interval. O There is a 98% chance that the confidence interval contains the sample mean time. O The confidence interval contains 98% of all sample times. 98% of all confidence intervals found using this same sampling technique will contain the population mean time. fastQUESTION 11 A manager wishes to simulate 10 days of operation of an inventory system. The beginning inventory at the start of the first day is 64 units. Demand is known to be uniformly distributed in In Exhibit 15-1, the production possibilities curves of wheat and corn for Nabia and Pada are presented. In Pada the cost of producing one more unit of corn is equal to:a. 1/3 unit of wheat.b. 3 units of wheat.c. 3 units of corn.d. 15 units of wheat.e. 30 units of wheat. How does the war in Ukrain cause the bullwip effect in supplychain with regrads to sunflower oil. x(t)=-ty(t)= t^2-4Graph the parametric equation and indicate the orientation. The following is a simple consumption function C = 500 + 0.7* (Y-T) Where Cis consumption, Y is income before tax, and T is taxes. Assume (in billions) 1 = 1,000 G = 500 T = 500 Determine (0) The multiplier value. Determine the equilibrium income without taxes. Determine the equilibrium income after the introduction of taxes.